Mail.ruПочтаМой МирОдноклассникиВКонтактеИгрыЗнакомстваНовостиКалендарьОблакоЗаметкиВсе проекты

Алгебра 8 класс

Анастасия Баюк Ученик (38), закрыт 1 год назад
Докажите, что (x^3+y^3+z^3)(x+y+z) <= 3(x^4+y^4+z^4) для всех неотрицательных x,y, z. Помогите, пожалуйста.
Лучший ответ
Арам и Абрам, братья Ивановы Гуру (2633) 1 год назад
Пусть например x ≥ y ≥ z ≥ 0 , в силу возрастания f(t)=t³ также x³ ≥ y³ ≥ z³
Тогда x⁴+y⁴+z⁴=x∙x³+y∙y³+z∙z³ в силу перестановочного неравенства есть максимальное из скалярных произведений вида
(y∙x³+z∙y³+x∙z³) и (z∙x³+x∙y³+y∙z³)
поэтому сумма 4-х степеней во всяком случае не меньше среднего арифметического:
x⁴+y⁴+z⁴≥ ⅓[(y∙x³+z∙y³+x∙z³)+(z∙x³+x∙y³+y∙z³)+(x∙x³+y∙y³+z∙z³)]=⅓(x+y+z)∙(x³+y³+z³)
Павел А. КоржовВысший разум (101975) 1 год назад
А что за пересиановочное неравенство? Вы утверждаете на его основе, что скалярка с (х,у,z) будет наибольшей из трех?
Арам и Абрам, братья Ивановы Гуру (2633) Павлентий Арлекинович Коржо, одно из моих любимых и часто используемых https://ru.wikipedia.org/wiki/Перестановочное_неравенство
Остальные ответы
Макс Человеч Гуру (3338) 1 год назад
Не буду, пусть поверят
Похожие вопросы